Difference between revisions of "2006 AMC 12A Problems/Problem 1"
Line 1: | Line 1: | ||
== Problem == | == Problem == | ||
+ | |||
+ | Sandwiches at Joe's Fast Food cost <math>$3</math> each and sodas cost <math>$2</math> each. How many dollars will it cost to purchase <math>5</math> sandwiches and <math>8</math> sodas? | ||
+ | |||
+ | <math> \mathrm{(A) \ } 31\qquad \mathrm{(B) \ } 32\qquad \mathrm{(C) \ } 33\qquad \mathrm{(D) \ } 34\qquad \mathrm{(E) \ } 35 </math> | ||
== Solution == | == Solution == |
Revision as of 22:34, 10 July 2006
Problem
Sandwiches at Joe's Fast Food cost each and sodas cost each. How many dollars will it cost to purchase sandwiches and sodas?